Supremum over integral of bounded and continuous functions is $+infty$Fubini's Theorem and bounded...

How do I prevent inappropriate ads from appearing in my game?

Why is participating in the European Parliamentary elections used as a threat?

I'm just a whisper. Who am I?

Unable to disable Microsoft Store in domain environment

Why would five hundred and five be same as one?

Isometric embedding of a genus g surface

Identifying "long and narrow" polygons in with PostGIS

Deciphering cause of death?

Why the "ls" command is showing the permissions of files in a FAT32 partition?

Animation: customize bounce interpolation

Language involving irrational number is not a CFL

What is this high flying aircraft over Pennsylvania?

The Digit Triangles

Why do Radio Buttons not fill the entire outer circle?

Do you waste sorcery points if you try to apply metamagic to a spell from a scroll but fail to cast it?

Make a Bowl of Alphabet Soup

Did I make a mistake by ccing email to boss to others?

Why is the Sun approximated as a black body at ~ 5800 K?

Mimic lecturing on blackboard, facing audience

When and why was runway 07/25 at Kai Tak removed?

Is there a way to play vibrato on the piano?

Can I say "fingers" when referring to toes?

Grepping string, but include all non-blank lines following each grep match

Why does the Persian emissary display a string of crowned skulls?



Supremum over integral of bounded and continuous functions is $+infty$


Fubini's Theorem and bounded functionsContinuity of a probability integral with supremumBounded Lipschitz Metric on Space of Positive MeasuresCountable additivity of measure with given “good” marginalsShow that $iint_{X times Y}varphi(x)k(x,y)psi(y) d(mu times nu)=int_Y Big[int_Xvarphi(x)k(x,y)dmu Big] psi(y) dnu$weak convergence and unbounded functions with bounded momentLink between convergence in Probability of a supremum and a limsupIs the set of all joint probability measure closed?Realize a coupling in the target space via a measure on the source spaceIs an ambiguity set with Wasserstein distance of order 1 is convex?













0












$begingroup$


Let $X, Y$ be Polish spaces with probability measures $mu, nu$, respectively; and let $gamma$ be a finite measure on the product $X times Y$. We consider the maximization problem
$$
sup_{varphi,psi} bigg{int_{X} varphi dmu + int_{Y} psi dnu - int_{X times Y} big( varphi(x) + psi(y) big) dgammabigg}
$$

over the set of all bounded and continuous functions $varphi(x),psi(y)$.



Why can this maximization problem always attain the value $+infty$ for every initial choices of $mu,nu,gamma$, if we exclude the case where $gamma$ has marginals $mu$ and $nu$?










share|cite|improve this question











$endgroup$












  • $begingroup$
    If $gamma$ required to be a probability measure?
    $endgroup$
    – kimchi lover
    Mar 12 at 22:32










  • $begingroup$
    $gamma$ is a finite measure on $X times Y$.
    $endgroup$
    – user63841219
    Mar 12 at 22:42
















0












$begingroup$


Let $X, Y$ be Polish spaces with probability measures $mu, nu$, respectively; and let $gamma$ be a finite measure on the product $X times Y$. We consider the maximization problem
$$
sup_{varphi,psi} bigg{int_{X} varphi dmu + int_{Y} psi dnu - int_{X times Y} big( varphi(x) + psi(y) big) dgammabigg}
$$

over the set of all bounded and continuous functions $varphi(x),psi(y)$.



Why can this maximization problem always attain the value $+infty$ for every initial choices of $mu,nu,gamma$, if we exclude the case where $gamma$ has marginals $mu$ and $nu$?










share|cite|improve this question











$endgroup$












  • $begingroup$
    If $gamma$ required to be a probability measure?
    $endgroup$
    – kimchi lover
    Mar 12 at 22:32










  • $begingroup$
    $gamma$ is a finite measure on $X times Y$.
    $endgroup$
    – user63841219
    Mar 12 at 22:42














0












0








0





$begingroup$


Let $X, Y$ be Polish spaces with probability measures $mu, nu$, respectively; and let $gamma$ be a finite measure on the product $X times Y$. We consider the maximization problem
$$
sup_{varphi,psi} bigg{int_{X} varphi dmu + int_{Y} psi dnu - int_{X times Y} big( varphi(x) + psi(y) big) dgammabigg}
$$

over the set of all bounded and continuous functions $varphi(x),psi(y)$.



Why can this maximization problem always attain the value $+infty$ for every initial choices of $mu,nu,gamma$, if we exclude the case where $gamma$ has marginals $mu$ and $nu$?










share|cite|improve this question











$endgroup$




Let $X, Y$ be Polish spaces with probability measures $mu, nu$, respectively; and let $gamma$ be a finite measure on the product $X times Y$. We consider the maximization problem
$$
sup_{varphi,psi} bigg{int_{X} varphi dmu + int_{Y} psi dnu - int_{X times Y} big( varphi(x) + psi(y) big) dgammabigg}
$$

over the set of all bounded and continuous functions $varphi(x),psi(y)$.



Why can this maximization problem always attain the value $+infty$ for every initial choices of $mu,nu,gamma$, if we exclude the case where $gamma$ has marginals $mu$ and $nu$?







real-analysis analysis probability-theory optimization supremum-and-infimum






share|cite|improve this question















share|cite|improve this question













share|cite|improve this question




share|cite|improve this question








edited Mar 12 at 22:41







user63841219

















asked Mar 12 at 22:09









user63841219user63841219

1468




1468












  • $begingroup$
    If $gamma$ required to be a probability measure?
    $endgroup$
    – kimchi lover
    Mar 12 at 22:32










  • $begingroup$
    $gamma$ is a finite measure on $X times Y$.
    $endgroup$
    – user63841219
    Mar 12 at 22:42


















  • $begingroup$
    If $gamma$ required to be a probability measure?
    $endgroup$
    – kimchi lover
    Mar 12 at 22:32










  • $begingroup$
    $gamma$ is a finite measure on $X times Y$.
    $endgroup$
    – user63841219
    Mar 12 at 22:42
















$begingroup$
If $gamma$ required to be a probability measure?
$endgroup$
– kimchi lover
Mar 12 at 22:32




$begingroup$
If $gamma$ required to be a probability measure?
$endgroup$
– kimchi lover
Mar 12 at 22:32












$begingroup$
$gamma$ is a finite measure on $X times Y$.
$endgroup$
– user63841219
Mar 12 at 22:42




$begingroup$
$gamma$ is a finite measure on $X times Y$.
$endgroup$
– user63841219
Mar 12 at 22:42










1 Answer
1






active

oldest

votes


















0












$begingroup$

Let $mu'$ and $nu'$ be the margins of $gamma$. At least one of $munemu'$ and $nunenu'$ must hold.
Your maximization problem becomes: maximize $$int_X varphi(dmu-dmu') +int_Ypsi(dnu-dnu').$$
Assume without loss of generality that $munemu'$, let the signed measure $sigma=mu-mu'.$ Then your maximization problem is lower bounded by the restricted maximization problem, of determining $$sup_varphi int_{X}varphi dsigma.$$ Since $sigma$ is not the zero measure, there exists a continuous bounded $f$ such that $int_X fsigma = 1$. Now it is clear that by taking $varphi$ to be large scalar multiples of $f$, you see the supremum is $infty$.






share|cite|improve this answer









$endgroup$













    Your Answer





    StackExchange.ifUsing("editor", function () {
    return StackExchange.using("mathjaxEditing", function () {
    StackExchange.MarkdownEditor.creationCallbacks.add(function (editor, postfix) {
    StackExchange.mathjaxEditing.prepareWmdForMathJax(editor, postfix, [["$", "$"], ["\\(","\\)"]]);
    });
    });
    }, "mathjax-editing");

    StackExchange.ready(function() {
    var channelOptions = {
    tags: "".split(" "),
    id: "69"
    };
    initTagRenderer("".split(" "), "".split(" "), channelOptions);

    StackExchange.using("externalEditor", function() {
    // Have to fire editor after snippets, if snippets enabled
    if (StackExchange.settings.snippets.snippetsEnabled) {
    StackExchange.using("snippets", function() {
    createEditor();
    });
    }
    else {
    createEditor();
    }
    });

    function createEditor() {
    StackExchange.prepareEditor({
    heartbeatType: 'answer',
    autoActivateHeartbeat: false,
    convertImagesToLinks: true,
    noModals: true,
    showLowRepImageUploadWarning: true,
    reputationToPostImages: 10,
    bindNavPrevention: true,
    postfix: "",
    imageUploader: {
    brandingHtml: "Powered by u003ca class="icon-imgur-white" href="https://imgur.com/"u003eu003c/au003e",
    contentPolicyHtml: "User contributions licensed under u003ca href="https://creativecommons.org/licenses/by-sa/3.0/"u003ecc by-sa 3.0 with attribution requiredu003c/au003e u003ca href="https://stackoverflow.com/legal/content-policy"u003e(content policy)u003c/au003e",
    allowUrls: true
    },
    noCode: true, onDemand: true,
    discardSelector: ".discard-answer"
    ,immediatelyShowMarkdownHelp:true
    });


    }
    });














    draft saved

    draft discarded


















    StackExchange.ready(
    function () {
    StackExchange.openid.initPostLogin('.new-post-login', 'https%3a%2f%2fmath.stackexchange.com%2fquestions%2f3145772%2fsupremum-over-integral-of-bounded-and-continuous-functions-is-infty%23new-answer', 'question_page');
    }
    );

    Post as a guest















    Required, but never shown

























    1 Answer
    1






    active

    oldest

    votes








    1 Answer
    1






    active

    oldest

    votes









    active

    oldest

    votes






    active

    oldest

    votes









    0












    $begingroup$

    Let $mu'$ and $nu'$ be the margins of $gamma$. At least one of $munemu'$ and $nunenu'$ must hold.
    Your maximization problem becomes: maximize $$int_X varphi(dmu-dmu') +int_Ypsi(dnu-dnu').$$
    Assume without loss of generality that $munemu'$, let the signed measure $sigma=mu-mu'.$ Then your maximization problem is lower bounded by the restricted maximization problem, of determining $$sup_varphi int_{X}varphi dsigma.$$ Since $sigma$ is not the zero measure, there exists a continuous bounded $f$ such that $int_X fsigma = 1$. Now it is clear that by taking $varphi$ to be large scalar multiples of $f$, you see the supremum is $infty$.






    share|cite|improve this answer









    $endgroup$


















      0












      $begingroup$

      Let $mu'$ and $nu'$ be the margins of $gamma$. At least one of $munemu'$ and $nunenu'$ must hold.
      Your maximization problem becomes: maximize $$int_X varphi(dmu-dmu') +int_Ypsi(dnu-dnu').$$
      Assume without loss of generality that $munemu'$, let the signed measure $sigma=mu-mu'.$ Then your maximization problem is lower bounded by the restricted maximization problem, of determining $$sup_varphi int_{X}varphi dsigma.$$ Since $sigma$ is not the zero measure, there exists a continuous bounded $f$ such that $int_X fsigma = 1$. Now it is clear that by taking $varphi$ to be large scalar multiples of $f$, you see the supremum is $infty$.






      share|cite|improve this answer









      $endgroup$
















        0












        0








        0





        $begingroup$

        Let $mu'$ and $nu'$ be the margins of $gamma$. At least one of $munemu'$ and $nunenu'$ must hold.
        Your maximization problem becomes: maximize $$int_X varphi(dmu-dmu') +int_Ypsi(dnu-dnu').$$
        Assume without loss of generality that $munemu'$, let the signed measure $sigma=mu-mu'.$ Then your maximization problem is lower bounded by the restricted maximization problem, of determining $$sup_varphi int_{X}varphi dsigma.$$ Since $sigma$ is not the zero measure, there exists a continuous bounded $f$ such that $int_X fsigma = 1$. Now it is clear that by taking $varphi$ to be large scalar multiples of $f$, you see the supremum is $infty$.






        share|cite|improve this answer









        $endgroup$



        Let $mu'$ and $nu'$ be the margins of $gamma$. At least one of $munemu'$ and $nunenu'$ must hold.
        Your maximization problem becomes: maximize $$int_X varphi(dmu-dmu') +int_Ypsi(dnu-dnu').$$
        Assume without loss of generality that $munemu'$, let the signed measure $sigma=mu-mu'.$ Then your maximization problem is lower bounded by the restricted maximization problem, of determining $$sup_varphi int_{X}varphi dsigma.$$ Since $sigma$ is not the zero measure, there exists a continuous bounded $f$ such that $int_X fsigma = 1$. Now it is clear that by taking $varphi$ to be large scalar multiples of $f$, you see the supremum is $infty$.







        share|cite|improve this answer












        share|cite|improve this answer



        share|cite|improve this answer










        answered Mar 12 at 23:08









        kimchi loverkimchi lover

        11.3k31229




        11.3k31229






























            draft saved

            draft discarded




















































            Thanks for contributing an answer to Mathematics Stack Exchange!


            • Please be sure to answer the question. Provide details and share your research!

            But avoid



            • Asking for help, clarification, or responding to other answers.

            • Making statements based on opinion; back them up with references or personal experience.


            Use MathJax to format equations. MathJax reference.


            To learn more, see our tips on writing great answers.




            draft saved


            draft discarded














            StackExchange.ready(
            function () {
            StackExchange.openid.initPostLogin('.new-post-login', 'https%3a%2f%2fmath.stackexchange.com%2fquestions%2f3145772%2fsupremum-over-integral-of-bounded-and-continuous-functions-is-infty%23new-answer', 'question_page');
            }
            );

            Post as a guest















            Required, but never shown





















































            Required, but never shown














            Required, but never shown












            Required, but never shown







            Required, but never shown

































            Required, but never shown














            Required, but never shown












            Required, but never shown







            Required, but never shown







            Popular posts from this blog

            Magento 2 - Add success message with knockout Planned maintenance scheduled April 23, 2019 at 23:30 UTC (7:30pm US/Eastern) Announcing the arrival of Valued Associate #679: Cesar Manara Unicorn Meta Zoo #1: Why another podcast?Success / Error message on ajax request$.widget is not a function when loading a homepage after add custom jQuery on custom themeHow can bind jQuery to current document in Magento 2 When template load by ajaxRedirect page using plugin in Magento 2Magento 2 - Update quantity and totals of cart page without page reload?Magento 2: Quote data not loaded on knockout checkoutMagento 2 : I need to change add to cart success message after adding product into cart through pluginMagento 2.2.5 How to add additional products to cart from new checkout step?Magento 2 Add error/success message with knockoutCan't validate Post Code on checkout page

            Fil:Tokke komm.svg

            Where did Arya get these scars? Unicorn Meta Zoo #1: Why another podcast? Announcing the arrival of Valued Associate #679: Cesar Manara Favourite questions and answers from the 1st quarter of 2019Why did Arya refuse to end it?Has the pronunciation of Arya Stark's name changed?Has Arya forgiven people?Why did Arya Stark lose her vision?Why can Arya still use the faces?Has the Narrow Sea become narrower?Does Arya Stark know how to make poisons outside of the House of Black and White?Why did Nymeria leave Arya?Why did Arya not kill the Lannister soldiers she encountered in the Riverlands?What is the current canonical age of Sansa, Bran and Arya Stark?